3
エンタメ解説
文献あり

中央二項係数っていつもそうですよね?私の事どう思っているんですか?

203
0
$$$$

中央ニ項係数って素敵♡

んちゃ!
今回は中央二項係数で作って遊ぶよ!
この記事は書きかけだよ!仕事しながらだから許して...

ベータ関数とに項係数を含む和

\begin{eqnarray} B(m,n)&=&\frac{(m-1)!(n-1)!)}{(m+n-1)!}\\ &=&\int_{0}^{1}x^{m-1}(1-x)^{n-1}dx\quad (m,n\in\mathbb{N}) \end{eqnarray}

\begin{equation} \begin{pmatrix}n\\k\end{pmatrix}^{-1}=(n+1)\int_{0}^{1}x^{k}(1-x)^{n-k}dx \end{equation}

Lehmerの仕事

\begin{eqnarray} \left\{ \begin{array}{l} \sum_{n=0}^{\infty}a_{n}\begin{pmatrix}2n\\n\end{pmatrix}\\ \sum_{n=0}^{\infty}\frac{a_{n}}{\begin{pmatrix}2n\\n\end{pmatrix}} \end{array} \right. \end{eqnarray}

\begin{eqnarray} &&y=(\arcsin{x})^{2}\\ &&y^{'}=\frac{2\arcsin{x}}{\sqrt{1-x^{2}}}\\ &&y^{''}=\frac{2}{1-x^{2}}(\frac{x\arcsin{x}}{\sqrt{1-x^{2}}}+1) \end{eqnarray}
ゆえに以下の微分方程式を得る。
\begin{equation} (1-x^{2})y^{''}-xy^{'}=2 \end{equation}

解析的解

$y=\sum_{n=0}^{\infty}a_{n}x^{n}$と置きこれを先の微分方程式に代入すると次の様に計算できる。
\begin{eqnarray} &&2a_{2}+\sum_{n=1}^{\infty}\{(n+2)(n+1)a_{n+2}-n^{2}a_{n}\}=2\\ &&a_{2}=1\\ &&a_{n+2}=\frac{n^{2}}{(n+2)(n+1)}a_{n} \end{eqnarray}
\begin{eqnarray} a_{2n}&=&\frac{(2n-2)^{2}}{2n(2n-1)}a_{2(n-1)}\\ &=&\frac{2^{2n-1}\{(n-1)!\}^{2}}{(2n)!}a_{2}\\ &=&\frac{2^{2n-1}(n!)^{2}}{n^{2}(2n)!}\\ &=&\frac{2^{2n-1}}{n^{2}\begin{pmatrix}2n\\n\end{pmatrix}} \end{eqnarray}
\begin{eqnarray} &&a_{0}=0\\ &&a_{1}=a_{3}=\cdots =0 \end{eqnarray}
\begin{equation} (\arcsin{x})^{2}=\frac{1}{2}\sum_{n=1}^{\infty}\frac{2^{2n}}{n^{2}\begin{pmatrix}2n\\n\end{pmatrix}}x^{2n} \end{equation}

\begin{eqnarray} \frac{1}{\begin{pmatrix}2n\\n\end{pmatrix}}=nB(n,n+1) \end{eqnarray}

\begin{eqnarray} \sum_{n=0}^{\infty}\frac{x^{n}}{\begin{pmatrix}2n\\n\end{pmatrix}}&=&\int_{0}^{1}\sum_{n=0}^{\infty}nx^{n}t^{n-1}(1-t)^{n}dt\\ &=&\int_{0}^{1}x(1-t)\sum_{n=0}^{\infty}n(xt(1-t))^{n-1}dt\\ &=&\int_{0}^{1}\frac{x(1-t)}{(1-xt(1-t))^{2}} \end{eqnarray}

\begin{equation} \sum_{n=0}^{\infty}\frac{x^{n}}{\begin{pmatrix}2n\\n\end{pmatrix}}=\frac{4\sqrt{x}}{(4-x)^{\frac{3}{2}}}\arctan{(\sqrt{\frac{x}{4-x}})}+\frac{x}{4-x}+1 \end{equation}

\begin{equation} \sum_{n=1}^{\infty}\frac{x^{n}}{n\begin{pmatrix}2n\\n\end{pmatrix}}=\sqrt{\frac{4x}{4-x}}\arctan{\sqrt{\frac{x}{4-x}}} \end{equation}

\begin{eqnarray} \sum_{n=1}^{\infty}\frac{x^{n}}{n\begin{pmatrix}2n\\n\end{pmatrix}}&=&\int_{0}^{1}\sum_{n=1}^{\infty}x^{n}t^{n-1}(1-t)^{n}dt\\ &=&\int_{0}^{1}x(1-t)\sum_{n=1}^{\infty}(xt(1-t))^{n-1}dt\\ &=&\int_{0}^{1}\frac{x(1-t)}{1-xt(1-t)}dt \end{eqnarray}
\begin{equation} xt^{2}-xt+1=x(t-\frac{1}{2})^{2}+\frac{4-x}{4} \end{equation}
\begin{eqnarray} \int_{0}^{1}\frac{x(1-t)}{1-xt(1-t)}dt&=&\int_{0}^{1}\frac{1-t}{(t-\frac{1}{2})^{2}+\frac{4-x}{4x}}dt \quad(t-\frac{1}{2}=\sqrt{\frac{4-x}{4x}}\tan{\theta})\\ &=&\sqrt{\frac{4x}{4-x}}\int_{-\arctan{\sqrt{\frac{x}{4-x}}}}^{\arctan{\sqrt{\frac{x}{4-x}}}}(\frac{1}{2}-\sqrt{\frac{4-x}{4x}}\tan{\theta})d\theta\\ &=&\sqrt{\frac{4x}{4-x}}\int_{0}^{\arctan{\sqrt{\frac{x}{4-x}}}}d\theta\\ &=&\sqrt{\frac{4x}{4-x}}\arctan{\sqrt{\frac{x}{4-x}}}\\ &=&2\sqrt{\frac{x}{4-x}}\arctan{\sqrt{\frac{x}{4-x}}} \end{eqnarray}

\begin{eqnarray} \frac{d}{dx}\sum_{n=1}^{\infty}\frac{x^{n}}{n\begin{pmatrix}2n\\n\end{pmatrix}}&=&\sum_{n=1}^{\infty}\frac{x^{n-1}}{\begin{pmatrix}2n\\n\end{pmatrix}}\\ &=&\frac{4}{(4-x)^{\frac{3}{2}}\sqrt{x}}\arctan{\sqrt{\frac{x}{4-x}}}+\frac{2}{4-x} \end{eqnarray}
\begin{equation} \sum_{n=0}^{\infty}\frac{x^{n}}{\begin{pmatrix}2n\\n\end{pmatrix}}=\frac{4\sqrt{x}}{(4-x)^{\frac{3}{2}}}\arctan{\sqrt{\frac{x}{4-x}}}+\frac{x}{4-x}+1 \end{equation}

$x=1$の場合
\begin{eqnarray} \left\{ \begin{array}{l} \sum_{n=1}^{\infty}\frac{1}{n\begin{pmatrix}2n\\n\end{pmatrix}}=\frac{\pi}{3\sqrt{3}}\\ \sum_{n=0}^{\infty}\frac{1}{\begin{pmatrix}2n\\n\end{pmatrix}}=\frac{2\sqrt{3}}{27}\pi+\frac{4}{3} \end{array} \right. \end{eqnarray}

\begin{equation} \sum_{k=0}^{\infty}\frac{x^{k}}{\begin{pmatrix}2k+1\\k\end{pmatrix}}=\frac{2}{4-x}+\frac{8}{(4-x)\sqrt{4x-x^{2}}}\arctan{\frac{x}{\sqrt{4x-x^{2}}}} \end{equation}

\begin{eqnarray} \sum_{k=0}^{\infty}\frac{x^{k}}{\begin{pmatrix}2k+1\\k\end{pmatrix}}&=&\int_{0}^{1}\sum_{k=0}^{\infty}(2k+2)x^{k}t^{k}(1-t)^{k+1}dt\\ &=&\frac{2}{xt}\int_{0}^{1}\sum_{k=0}^{\infty}k(xt(1-t))^{k}dt\\ &=&\frac{2}{xt}\int_{0}^{1}\frac{xt(1-t)}{(1-xt(1-t))^{2}}dt\\ &=&2\int_{0}^{1}\frac{1-t}{(1-xt(1-t))^{2}}dt \quad(1-t=\tau)\\ &=&2\int_{0}^{1}\frac{\tau}{(1-x\tau(1-\tau))^{2}}d\tau \quad(\tau-\frac{1}{2}=\sqrt{\frac{4-x}{4x}}\tan\theta)\\ &=&\frac{2}{x^{2}}\frac{16x^{2}}{(4-x)^{2}}\sqrt{\frac{4-x}{4x}}\int_{-\arctan{\sqrt{\frac{x}{4-x}}}}^{\arctan{\sqrt{\frac{x}{4-x}}}}(\sqrt{\frac{4-x}{4x}}\tan\theta+\frac{1}{2})\cos^{2}\theta d\theta\\ &=&\frac{16}{\sqrt{x}(4-x)^{\frac{3}{2}}}\int_{0}^{\arctan{\sqrt{\frac{x}{4-x}}}}\cos^{2}\theta d\theta\\ &=&\frac{8}{\sqrt{x}(4-x)^{\frac{3}{2}}}\arctan{\sqrt{\frac{x}{4-x}}}+\frac{2}{4-x}\\ &=&\frac{8}{(4-x)\sqrt{4x-x^{2}}}\arctan{\frac{x}{\sqrt{4x-x^{2}}}}+\frac{2}{4-x}\quad(0\lt x\lt 4)\\ &=&\frac{4}{(4-x)\sqrt{-4x+x^{2}}}\log{\frac{1-\sqrt{-4x+x^{2}}}{1+\sqrt{-4x+x^{2}}}}\quad(-4\lt x\lt 0) \end{eqnarray}
最後の変形は次の事実を用いた。
\begin{eqnarray} \left\{ \begin{array}{l} \theta=\arctan\lambda\\ \sin\theta=\frac{\lambda}{\sqrt{1+\lambda^{2}}}\\ \cos\theta=\frac{1}{\sqrt{1+\lambda^{2}}}\\ \frac{1}{i}\arctan{\frac{\theta}{i}}=\frac{1}{2}\log{\frac{1-\theta}{1+\theta}} \end{array} \right. \end{eqnarray}

\begin{equation} \sum_{k=0}^{\infty}\frac{x^{k}}{\begin{pmatrix}2k+1\\k\end{pmatrix}}=D \end{equation}

\begin{equation} \sum_{m=0}^{\infty}\frac{x^{m}}{\begin{pmatrix}2m+1\\m\end{pmatrix}(2m+3)}=(\frac{4}{x}-1)D-\frac{4}{x} \end{equation}

実際に計算して係数比較するだけ。
\begin{equation} 4(k+1)(k+2)-(2k+3)(2k+2)=2(k+1) \end{equation}

$f(x)=\sum_{n=0}^{\infty}a_{n}x^{n}$とすると次の事が成り立つ。
\begin{equation} \sum_{n=1}^{\infty}\frac{a_{n-1}x^{n}}{n\begin{pmatrix}2n\\n\end{pmatrix}}=\int_{0}^{1}x(1-t)f(xt(1-t))dt \end{equation}

$S(k;x)=\sum_{n=0}n^{k}x^{n}$とすると次の様に書ける。
\begin{equation} \sum_{n=0}^{\infty}\frac{n^{k}}{\begin{pmatrix}2n\\n\end{pmatrix}}=2\int_{0}^{1}S(k+1;x(1-x))dx+\int_{0}^{1}S(k;x(1-x))dx \end{equation}

\begin{eqnarray} \sum_{n=0}^{\infty}\frac{n^{k}}{\begin{pmatrix}2n\\n\end{pmatrix}}&=&\sum_{n=0}^{\infty}\frac{n^{k}n!n!}{(2n)!}\\ &=&\sum_{n=0}^{\infty}\frac{n^{k}\Gamma{(n+1)}\Gamma{(n+1)}}{\Gamma{(2n+1)}}\\ &=&\sum_{n=0}^{\infty}n^{k}(2n+1)B(n+1,n+1)\\ &=&2\int_{0}^{1}\sum_{n=0}^{\infty}n^{k+1}x^{n}(1-x)^{n}dx+\int_{0}^{1}\sum_{n=0}^{\infty}n^{k}x^{n}(1-x)^{n}dx\\ &=&2\int_{0}^{1}S(k+1;x(1-x))dx+\int_{0}^{1}S(k;x(1-x))dx \end{eqnarray}

\begin{equation} S(k;x)=\sum_{n=0}^{\infty}n^{k}x^{n}\Rightarrow x\frac{d}{dx}S(k;x)=S(k+1;x) \end{equation}

\begin{eqnarray} x\frac{d}{dx}S(k;x)&=&x\frac{d}{dx}\sum_{n=0}^{\infty}n^{k}x^{n}\\ &=&x\sum_{n=0}^{\infty}n^{k+1}x^{n-1}\\ &=&\sum_{n=0}^{\infty}n^{k+1}x^{n}\\ &=&S(k+1;x) \end{eqnarray}

\begin{eqnarray} S(k;x)&=&(x\frac{d}{dx})^{k}S(0;x)\\ &=&(x\frac{d}{dx})^{k}\frac{1}{1-x} \end{eqnarray}

$(x\frac{d}{dx})^{k}$$\frac{d^{i}}{dx^{i}}\quad(i=1,2,...,k)$の係数は次の様に書ける。
ただし、第$k$行、第$i$列は第$k-1$行の第$i-1,i$列の$1,i$倍して足し合わせたもの。

1
11
131
1761
11525101

$k=0$とすると次の事が成り立つ。
\begin{eqnarray} \sum_{n=0}^{\infty}\frac{1}{\begin{pmatrix}2n\\n\end{pmatrix}}&=&2\int_{0}^{1}\frac{x(1-x)}{(1-x(1-x))^{2}}dx+\int_{0}^{1}\frac{1}{1-x(1-x)}dx\\ &=&2\int_{0}^{1}\frac{1}{(1-x(1-x))^{2}}dx-\int_{0}^{1}\frac{1}{1-x(1-x)}dx\\ &=&\frac{8\sqrt{3}}{27}\pi-\frac{2\sqrt{3}}{9}\pi\\ &=&\frac{2\sqrt{3}}{27}\pi+\frac{4}{3} \end{eqnarray}

\begin{equation} \sum_{k=0}^{n}\frac{a^{k}b^{n-k}}{\begin{pmatrix}n\\k\end{pmatrix}}=\frac{n+1}{(a+b)(\frac{1}{a}+\frac{1}{b})^{n}}\sum_{k=1}^{n+1}\frac{(a^{k}+b^{k})(\frac{1}{a}+\frac{1}{b})^{k-1}}{k} \end{equation}

\begin{eqnarray} \sum_{n=0}^{\infty}x^{n}\sum_{k=0}^{n}\frac{a^{k}b^{n-k}}{\begin{pmatrix}n\\k\end{pmatrix}}&=&\sum_{n=0}^{\infty}\sum_{k=0}^{n}(n+1)x^{n}a^{k}b^{n-k}\int_{0}^{1}t^{k}(1-t)^{n-k}dt\\ &=&\frac{d}{dx}(x\int_{0}^{1}\sum_{k=0}^{\infty}\sum_{n=k}^{\infty}(axt)^{k}(bx(1-t))^{n-k}dt)\\ &=&\frac{d}{dx}(x\int_{0}^{1}\frac{1}{1-axt}\frac{1}{1-bx(1-t)})dt\\ &=&\frac{d}{dx}(\frac{1}{a+b-abx}\int_{0}^{1}(\frac{ax}{1-axt}+\frac{bx}{1-bx+bxt})dt)\\ &=&\frac{d}{dx}(\frac{-\log{(1-ax)}-\log{(1-bx)}}{a+b-abx})\\ &=&\frac{d}{dx}[\frac{1}{a+b}\sum_{n=0}^{\infty}\frac{a^{n}b^{n}x^{n}}{(a+b)^{n}}\sum_{k=1}^{\infty}\frac{a^{k}+b^{k}}{k}x^{k}]\\ &=&\frac{d}{dx}[x\frac{1}{a+b}\sum_{n=0}^{\infty}\frac{a^{n}b^{n}x^{n}}{(a+b)^{n}}\sum_{k=1}^{\infty}\frac{a^{k}+b^{k}}{k}x^{k-1}]\\ &=&\frac{d}{dx}[x\frac{1}{a+b}\sum_{n=0}^{\infty}\frac{a^{n}b^{n}x^{n}}{(a+b)^{n}}\sum_{k=0}^{\infty}\frac{a^{k+1}+b^{k+1}}{k+1}x^{k}]\\ &=&\frac{d}{dx}[x\frac{1}{a+b}\sum_{\lambda=0}^{\infty}x^{\lambda}\sum_{k=0}^{\lambda}\frac{a^{k}b^{k}}{(a+b)^{k}}\frac{a^{\lambda-k+1}+b^{\lambda-k+1}}{\lambda-k+1}]\\ &=&\frac{d}{dx}[\frac{1}{a+b}\sum_{\lambda=0}^{\infty}x^{\lambda+1}\sum_{k=0}^{\lambda}\frac{a^{\lambda-k}b^{\lambda-k}}{(a+b)^{\lambda-k}}\frac{a^{k+1}+b^{k+1}}{k+1}]\\ &=&\frac{d}{dx}[\frac{1}{a+b}\sum_{n=0}^{\infty}\frac{a^{n}b^{n}x^{n+1}}{(a+b)^{n}}\sum_{k=0}^{n}\frac{a^{k+1}+b^{k+1}}{k+1}\frac{(a+b)^{k}}{a^{k}b^{k}}]\\ &=&\frac{1}{a+b}\sum_{n=0}^{\infty}\frac{(n+1)a^{n}b^{n}x^{n}}{(a+b)^{n}}\sum_{k=1}^{n+1}\frac{a^{k}+b^{k}}{k}\frac{(a+b)^{k-1}}{a^{k-1}b^{k-1}} \end{eqnarray}
$x$の各次数の係数比較を行うと次に結論を得る。
\begin{equation} \sum_{k=0}^{n}\frac{a^{k}b^{n-k}}{\begin{pmatrix}n\\k\end{pmatrix}}=\frac{n+1}{(a+b)(\frac{1}{a}+\frac{1}{b})^{n}}\sum_{k=1}^{n+1}\frac{(a^{k}+b^{k})(\frac{1}{a}+\frac{1}{b})^{k-1}}{k} \end{equation}

$a=b=1$とすると次式が成り立つ。
\begin{eqnarray} \sum_{k=0}^{n}\frac{1}{\begin{pmatrix}n\\k\end{pmatrix}}&=&\frac{n+1}{2^{n+1}}\sum_{k=1}^{n+1}\frac{2^{k}}{k}\\ &=&(n+1)\sum_{k=0}^{n}\frac{1}{(n+1-k)2^{k}} \end{eqnarray}

$a=x,b=1$とおくと次式を得る。
\begin{eqnarray} \sum_{k=0}^{n}\frac{x^{k}}{\begin{pmatrix}n\\k\end{pmatrix}}&=&\frac{n+1}{(1+z)}(\frac{z}{1+z})^{n}\sum_{k=1}^{n+1}\frac{1+z^{k}}{k}(\frac{1+z}{z})^{k} \end{eqnarray}

逆べき生成変換

次の変換を逆べき生成変換と呼ぶことにする。
\begin{equation} I(f|x)=\int_{0}^{x}\frac{f(t)-f(0)}{t}dt \end{equation}
また、$n\in\mathbb{N}$に対して次の様な変換を定義する。これを$n$階逆べき生成変換と呼ぶことにする。
\begin{equation} I^{n}(f|x)=\int_{0}^{x}dx_{1}\int_{0}^{x_{1}}dx_{2}\cdots\int_{0}^{x_{n-1}}dx_{n}\frac{f(x_{n})-f(0)}{x_{n}} \end{equation}

$f(x)=\sum_{n=0}^{\infty}a_{n}x^{n}$$k$回逆べき生成変換を行った結果は次の様に書ける。
\begin{equation} I^{k}(f|x)=\sum_{n=1}^{\infty}\frac{a_{n}}{n^{k}}x^{n} \end{equation}

q級数とに項係数を含む総和

q級数のEuler展開

\begin{equation} \Pi_{k=1}^{\infty}(1-q^{k})=1+\sum_{k=1}^{\infty}(-1)^{k}(q^{\frac{k(3k-1)}{2}}+q^{\frac{k(3k+1)}{2}}) \end{equation}

[1]$f(q;x)=1-\sum_{k=1}^{\infty}q^{k}x^{k+1}\Pi_{l=1}^{k-1}(1-q^{l}x)$と置く。するとこれは次の式を満たす。
\begin{equation} f(q;x)=1-qx^{2}-q^{2}x^{3}f(q;qx) \end{equation}
これは次の様にして証明できる。
\begin{eqnarray} f(q;x)&=&1-qx^{2}-\sum_{k=2}^{\infty}q^{k}x^{k+1}\Pi_{l=1}^{k-1}(1-q^{l}x)\\ &=&1-qx^{2}-\sum_{k=1}^{\infty}q^{k+1}x^{k+2}\Pi_{l=1}^{k}(1-q^{l}x)\\ &=&1-qx^{2}-\sum_{k=1}^{\infty}q^{k+1}x^{k+2}\Pi_{l=2}^{k}(1-q^{l}x)+\sum_{k=1}^{\infty}q^{k+2}x^{k+3}\Pi_{l=2}^{k}(1-q^{l}x)\\ &=&1-qx^{2}-q^{2}x^{3}(1+\sum_{k=1}^{\infty}q^{k}x^{k}(\Pi_{l=2}^{k+1}(1-q^{l}x)-\Pi_{l=2}^{k}(1-q^{l}x)))\\ &=&1-qx^{2}-q^{2}x^{3}(1-\sum_{k=1}^{\infty}q^{k}(qx)^{k+1}\Pi_{l=2}^{k}(1-q^{l-1}(qx)))\\ &=&1-qx^{2}-q^{2}x^{3}(1-\sum_{k=1}^{\infty}q^{k}(qx)^{k+1}\Pi_{l=1}^{k-1}(1-q^{l}(qx)))\\ &=&1-qx^{2}-q^{2}x^{3}f(q;qx) \end{eqnarray}
[2]
\begin{eqnarray} f(q;x)&=&1-qx^{2}-q^{2}x^{3}f(q;qx)\\ &=&1-qx^{2}-q^{2}x^{3}(1-q^{3}x^{2}-q^{5}x^{3}f(q;q^{2}x))\\ &=&1-qx^{2}-q^{2}x^{3}+q^{5}x^{5}+q^{7}x^{6}f(q;q^{2}x)\\ &=&1-qx^{2}-q^{2}x^{3}+q^{5}x^{5}+q^{7}x^{6}(1-q^{5}x^{2}-q^{8}x^{3}f(q;q^{3}x))\\ &=&1-qx^{2}-q^{2}x^{3}+q^{5}x^{5}+q^{7}x^{6}-q^{12}x^{8}-q^{15}x^{9}f(q;q^{3}x) \end{eqnarray}
ここで少し考察をしてみる。とりあえず符号は抜きにして、$qx^{2},q^{5}x^{5},q^{12}x^{8},...;q^{2}x^{3},q^{7}x^{6},q^{15}x^{9},...$のべきをテーブルとして並べてみる。

(1,2)(2,3)
(5,5)(7,6)
(12,8)(15,9)
このテーブルを見るとわかると思うが、前者は$(\frac{n(3n-1)}{2},3n-1)$後者$(\frac{n(3n+1)}{2},3n)$なる規則で並んでいることが分かる。よって、これを示す。
[3]
これを示すには、上記方法を$N$回使用した際に出てくる$f(q;q^{N}x)$にかけられている$q^{\mu}x^{\nu}$$\mu,\nu$を求めればよい。
$f(q;q^{N}x)$だけに着目し符号を無視すると、$q^{2}x^{3}q^{2}(qx)^{3}\cdots q^{2}(q^{N-1}x)^{3}=q^{\frac{N}{2}(3N+1)}x^{3N}$
ゆえに、$q^{2}x^{3},q^{7}x^{6},q^{15}x^{9},...$のグループのべきの組は$(\frac{N}{2}(3N+1),3N)$また、$qx^{2},q^{5}x^{5},q^{12}x^{8},...$のグループの場合は$f(q,x)$を始まりとして先の結果に$q(q^{N-1}x)^{2}=q^{2N-1}x^{2}$をかけたものなので、$\frac{N-1}{2}(3N-2)+2N-1=\frac{N(3N-1)}{2}$
[4]
よって結論
\begin{eqnarray} f(q;x)&=&1+\sum_{n=1}^{\infty}(-1)^{n}(q^{\frac{n(3n-1)}{2}}x^{3n-1}+q^{\frac{n(3n+1)}{2}}x^{3n})\\ \end{eqnarray}
[5]
帰納法を使うと次の式を簡単に証明できる。
\begin{equation} 1-\sum_{n=1}^{N}\Pi_{k=1}^{n-1}(1-q^{k})q^{n}=\Pi_{n=1}^{N}(1-q^{n}) \end{equation}
[6]
[5]より$x=1$とすれば次の式が成り立つ。
\begin{eqnarray} f(q;1)&=&\Pi_{n=1}^{\infty}(1-q^{n})\\ &=&\sum_{n=-\infty}^{\infty}(-1)^{n}q^{\frac{n(3n+1)}{2}} \end{eqnarray}

$(\alpha)_{n}=\Pi_{k=0}^{n-1}(1-q^{k-1}a)$

\begin{equation} (\alpha)_{n}=\frac{(\alpha)_{\infty}}{(q^{n}\alpha)_{\infty}} \end{equation}

q-超幾何級数

\begin{equation} {}_{2}\psi_{1}{(\alpha,\beta;\gamma|x)}=\sum_{n=0}^{\infty}\frac{(\alpha)_{n}(\beta)_{n}}{(\gamma)_{n}(q)_{n}}x^{n} \end{equation}

\begin{equation} \frac{(az)_{\infty}}{(z)_{\infty}}=\sum_{m=0}^{\infty}\frac{(\alpha)_{m}}{(c)_{m}}z^{m} \end{equation}

Jackson積分

\begin{equation} \int_{0}^{c}f(t)d_{q}t=c(1-q)\sum_{n=0}^{\infty}f(cq^{n})q^{n} \end{equation}

\begin{equation} \sum_{k=0}^{\infty}\begin{pmatrix}x\\k\end{pmatrix}\begin{pmatrix}y\\n-k\end{pmatrix}=\begin{pmatrix}x+y\\n\end{pmatrix} \end{equation}

$x+y$個の異なるものをn個選ぶ方法の数は$x$個のものA、$y$個の異なるものBに分け、Aから$k$個、Bから$n-k$個選択する方法の数で分け、$k=0,1,...,n$すべてに渡り総和を取ったものになるので証明完了。

\begin{equation} \sum_{k=0}^{n}\begin{pmatrix}n\\k\end{pmatrix}\begin{pmatrix}x+y-n\\x-k\end{pmatrix}=\begin{pmatrix}x+y\\x\end{pmatrix} \end{equation}

\begin{eqnarray} \sum_{k=0}^{n}\begin{pmatrix}n\\k\end{pmatrix}(-1)^{k}k^{m}= \left\{ \begin{array}{l} 0\quad if \ \ m\lt n\\ (-1)^{n}n!\quad if \ \ m=n \end{array} \right. \end{eqnarray}

$f(x)=(1-x)^{n}$とする。するとこの式は次の様に書ける。
\begin{equation} f(x)=\sum_{k=0}^{n}(-1)^{k}\begin{pmatrix}n\\k\end{pmatrix}x^{k} \end{equation}
\begin{equation} (x\frac{d}{dx})^{m}f(x)=\sum_{k=0}^{n}(-1)^{k}\begin{pmatrix}n\\k\end{pmatrix}k^{m}x^{k} \end{equation}
定理15より$(x\frac{d}{dx})^{m}=1+\cdots+x^{m}\frac{d^{m}}{dx^{m}}$とかけるので、$m\lt n$の場合は$(\frac{d}{dx})^{m}f(x)$$1-x$で割り切れるので$x=1$を代入すれば0。$m=n$の場合は、$1-x$で割り切れない項は$(-1)^{n}n!$のみ。
以上より証明は完了した。

\begin{eqnarray} \sum_{k=0}^{n}\begin{pmatrix}n\\k\end{pmatrix}(-1)^{k}(xk+y)^{m}= \left\{ \begin{array}{l} 0\quad if \ \ m\lt n\\ (-1)^{n}x^{n}n!\quad if \ \ m=n \end{array} \right. \end{eqnarray}

$\sum$の中の$k$の最大べきにのみ着目すればいい。$k$の最大べきが$n$未満の場合はすべて$0$
$m=n$の場合は$k^{n}$部分だけが0でない値を持つ。ゆえにその値は$(-1)^{n}x^{n}n!$

$f(t)=c_{0}+c_{1}t+\cdots+c_{m}t^{m}$の場合
\begin{eqnarray} \sum_{k=0}^{n}\begin{pmatrix}n\\k\end{pmatrix}(-1)^{k}f(k)^{m}= \left\{ \begin{array}{l} 0\quad if \ \ m\lt n\\ (-1)^{n}c_{n}n!\quad if \ \ m=n \end{array} \right. \end{eqnarray}

超幾何関数と二項係数

$x=0$近傍の超幾何関数は下記の様に定義される。
\begin{equation} y={}_{2}F(\alpha,\beta;\gamma|x)=\sum_{n=0}^{\infty}\frac{(\alpha)_{n}(\beta)_{n}}{(\gamma)_{n}n!}x^{n}\quad (|x|\lt 1) \end{equation}

$\partial=\frac{d}{dx},D=x\partial$とすると下記の式が成り立つ。
\begin{equation} ((D+a)(D+b)-\frac{D(D+c-1)}{x})y=0 \end{equation}

$D^{2}=x^{2}\partial^{2}+x\partial$より
\begin{eqnarray} (D+a)(D+b)&=&D^{2}+(a+b)D+ab\\ &=&x^{2}\partial^{2}+(a+b+1)x\partial+ab \end{eqnarray}
\begin{eqnarray} D(D+c-1)&=&x^{2}\partial^{2}+c x\partial \end{eqnarray}
より
\begin{eqnarray} ((D+a)(D+b)-\frac{D(D+c-1)}{x}){}_{2}F_{1}(a,b;c|x)&=&(x(x-1)\partial^{2}+((a+b+1)x-c)\partial+ab)\sum_{n=0}^{\infty}\frac{(a)_{n}(b)_{n}}{(c)_{n}n!}x^{n}\\ &=&\sum_{n=0}^{\infty}[-((n+1)n+(n+1)c)\frac{(a+n)(b+n)}{(c+n)(n+1)}+(n(n-1)+(a+b+1)n+ab)]\frac{(a)_{n}(b)_{n}}{(c)_{n}n!}x^{n}\\ &=&0 \end{eqnarray}

超幾何微分方程式

\begin{equation} (x(1-x)\partial^{2}+(c-(a+b+1)x)\partial-ab)y=0 \end{equation}

両辺を$x(1-x)$で割ると
\begin{eqnarray} (\partial^{2}+(\frac{c}{x(1-x)}-\frac{a+b+1}{1-x})\partial-ab)y&=&(\partial^{2}+(\frac{c}{x}-\frac{1+a+b-c}{1-x})\partial-\frac{ab}{x(1-x)})y\\ &=&0 \end{eqnarray}
より$x=1$の近傍での解は$e=1+a+b-c$とおけば${}_{2}F_{1}(a,b;e|1-x)$
これを示したいなら$1-x\rightarrow x$なる変数変換を行えばいい。

超幾何関数$y={}_{2}F_{1}(a,b;c|x)$は次の微分方程式を満たす。
$\psi{(x)}=x^{c}(1-x)^{e}\quad (e=1+a+b-c)$とすると次式が成り立つ。
\begin{equation} (\partial\psi(x)\partial-ab\frac{\psi{(x)}}{x(1-x)})y=0 \end{equation}

\begin{equation} \partial f(x)=f(x)\partial+f^{'}(x) \end{equation}
\begin{eqnarray} \partial\psi{(x)}&=&(x^{c}\partial+cx^{c-1})(1-x)^{e}\\ &=&x^{c}((1-x)^{e}\partial-e(1-x)^{e-1})+cx^{c-1}(1-x)^{e}\\ &=&\psi{(x)}(\partial-\frac{e}{1-x}+\frac{c}{x}) \end{eqnarray}
これを代入すると
\begin{eqnarray} (\partial\psi(x)\partial-ab\frac{\psi{(x)}}{x(1-x)})y&=&\psi{(x)}(\partial^{2}+(\frac{c}{x}-\frac{e}{1-x})\partial-\frac{ab}{x(1-x)})y\\ &=&0 \end{eqnarray}

$\mathscr{D}=\partial f(x)-g(x)$とし、$\mathscr{D}y=0$の解を$F(x)$と置く。

$F(z(x))$の満たす微分方程式は下記の様に書ける。
\begin{equation} (\partial f(z(x))\frac{\partial}{z^{'}(x)}-z^{'}(x)g(z(x)))F(z(x))=0 \end{equation}

\begin{eqnarray} \partial_{z}=\frac{1}{z^{'}(x)}\partial \end{eqnarray}
より
\begin{eqnarray} (\partial f(z(x))\frac{\partial}{z^{'}(x)}-z^{'}(x)g(z(x)))F(z(x))&=&z^{'}(x)(\partial_{z}f(z)\partial_{z}-g(z))F(z)\\ &=&0 \end{eqnarray}

$\psi{(x)}=x^{c}(1-x)^{e}\quad (e=1+a+b-c)$
\begin{equation} (\partial\psi(x)\partial-ab\frac{\psi{(x)}}{x(1-x)})y=0\quad(y(0)=1,y^{'}(0)=\frac{ab}{c}) \end{equation}
に対して$z(x)=x^{s}$とすると次式が成り立つ。
\begin{equation} (\partial x^{cs-s+1}(1-x^{s})^{e}\partial-s^{2}ab x^{cs-1}(1-x^{s})^{e-1})y(x^{s})=0 \end{equation}

$z(x)=\frac{1-x}{1+(r-1)x}$とすると以下の計算が成り立つ。
\begin{eqnarray} &&1-z=\frac{rx}{1+(r-1)x}\\ &&z^{'}(x)=-\frac{r}{(1+(r-1)x)^{2}} \end{eqnarray}
\begin{eqnarray} -\partial(x^{e}(1-x)^{c})\frac{r^{e-1}}{(1+(r-1)x)^{c+e-2}}\partial+\alpha\beta x^{e-1}(1-x)^{c-1}\frac{r^{e}}{(1+(r-1)x)^{c+e}}&=&-r^{e-1}(\partial(x^{e}(1-x)^{c})\frac{1}{(1+(r-1)x)^{c+e-2}}\partial-\alpha\beta rx^{e-1}(1-x)^{c-1}\frac{1}{(1+(r-1)x)^{c+e}})\\ \end{eqnarray}
これより、以下の式が成り立つ。
\begin{eqnarray} &&\rho(x)=\frac{1}{1+(r-1)x}\\ &&(\partial x^{e}(1-x)^{c}\rho^{c+e-2}(x)\partial-rab x^{e-1}(1-x)^{c-1}\rho^{c+e}{(x)})y=0 \end{eqnarray}

$z(x)=(\frac{1-x}{1+(r-1)x})^{s}=(\frac{1-x}{\rho{(x)}})^{s}$とすると、$z^{'}(x)=-rs\frac{(1-x)^{s-1}}{\rho^{s+1}(x)}$ゆえに次の微分方程式を得る。
$(r,s)=(2,2),(3,3),(4,2)$
\begin{eqnarray} \left\{ \begin{array}{l} (1+x)^{2}-(1-x)^{2}=4x\rightarrow\frac{4x}{(1-x)^{2}}\\ (1+2x)^{3}-(1-x)^{3}=9x\frac{1-x^{3}}{1-x}\rightarrow\frac{9x(1-x)}{(1-x)(1+2x)^{2}}\\ (1+3x)^{2}-(1-x)^{2}=8x\frac{1-x^{2}}{1-x}\rightarrow\frac{8x(1-x^{2})}{(1-x)(1+3x)^{2}} \end{array} \right. \end{eqnarray}

$\mathscr{D}_{i}=\partial f_{i}(x)\partial -g_{i}(x)\quad(i=1,2)$に対する解を$F_{1},F_{2}$とする。このとき、ある適当な関数$h$が存在して次の事が成り立つとする。
\begin{equation} h(x)F_{2}(x)=F_{1}(x) \end{equation}
このとき、$h\mathscr{D}_{1}h=\mathscr{D}_{2}$が成り立つとすると
\begin{eqnarray} \left\{ \begin{array}{l} f_{2}(x)=f_{1}(x)h^{2}(x)\\ g_{2}(x)=g_{1}(x)h^{2}(x)-(f_{1}(x)h^{'}(x))^{'}h(x) \end{array} \right. \end{eqnarray}

$\partial h-h\partial=h^{'}$を用いる。
\begin{eqnarray} h\partial f_{1}\partial h&=&(\partial h-h^{'})f_{1}(h\partial+h^{'})\\ &=&\partial f_{1}h^{2}\partial+\partial f_{1}hh^{'}-f_{1}hh^{'}\partial-f_{1}(h^{'})^{2}\\ &=&\partial f_{1}h^{2}\partial+(f_{1}h^{'})^{'}h \end{eqnarray}
ゆえに
\begin{eqnarray} h\mathscr{D}_{1}h&=&\partial f_{1}h^{2}\partial+(f_{1}h^{'})^{'}h-g_{1}h^{2}\\ &=&\partial f_{1}h^{2}\partial-(g_{1}h^{2}-(f_{1}h^{'})^{'}h) \end{eqnarray}

上記hが存在するとすると下記の式が成り立つ。
\begin{eqnarray} \mathscr{D}_{1}h(x)F_{2}(x)&=&\frac{1}{h(x)}\mathscr{D}_{2}F_{2}(x)\\ &=&0 \end{eqnarray}

一次変換

\begin{eqnarray} \left\{ \begin{array}{l} (1-x)^{a+b-c}{}_{2}F_{1}(a,b;c|x)={}_{2}F_{1}(c-a,c-b;c|x)\\ (1-x)^{a}{}_{2}F_{1}(a,b;c|x)={}_{2}F_{1}(a,c-b;c|\frac{x}{x-1}) \end{array} \right. \end{eqnarray}

[1]次の様に記号を決める。
\begin{eqnarray} \left\{ \begin{array}{l} h=(1-x)^{a+b-c}\\ F_{1}={}_{2}F_{1}(c-a,c-b;c|x)\\ F_{2}={}_{2}F_{1}(a,b;c|x) \end{array} \right. \end{eqnarray}
する
\begin{eqnarray} \left\{ \begin{array}{l} \mathscr{D}_{1}=\partial x^{c}(1-x)^{c-a-b+1}\partial-(c-a)(c-b)x^{c-1}(1-x)^{c-a-b}\\ \mathscr{D}_{2}=\partial x^{c}(1-x)^{a+b-c+1}\partial-abx^{c-1}(1-x)^{a+b-c} \end{array} \right. \end{eqnarray}
\begin{eqnarray} f_{1}h^{2}&=&x^{c}(1-x)^{c-a-b+1}(1-x)^{2a+2b-2c}\\ &=&x^{c}(1-x)^{a+b-c+1} \end{eqnarray}
\begin{eqnarray} g_{1}h^{2}-(f_{1}h^{'})^{'}h&=&(c-a)(c-b)x^{c-1}(1-x)^{a+b-c}+(a+b-c)cx^{c-1}(1-x)^{a+b-c}\\ &=&abx^{c-1}(1-x)^{a+b-c}\\ &=&g_{2} \end{eqnarray}
初期条件検査
\begin{eqnarray} \left\{ \begin{array}{l} F_{1}(0)=(hF_{2})(0)=1\\ F^{'}_{1}(0)=(hF_{2})^{'}(0)=\frac{(c-a)(c-b)}{c} \end{array} \right. \end{eqnarray}
[2]次の様に記号を定める。
\begin{eqnarray} \left\{ \begin{array}{l} h=(1-x)^{a}\\ F_{1}={}_{2}F_{1}(a,c-b;c|\frac{x}{x-1})\\ F_{2}={}_{2}F_{1}(a,b;c|x) \end{array} \right. \end{eqnarray}
\begin{eqnarray} \left\{ \begin{array}{l} \mathscr{D}_{1}=\partial x^{c}(1-x)^{-a+b-c+1}\partial+a(c-b)x^{c-1}(1-x)^{-a+b-c-1}\\ \mathscr{D}_{2}=\partial x^{c}(1-x)^{a+b-c+1}\partial-abx^{c-1}(1-x)^{a+b-c} \end{array} \right. \end{eqnarray}
以上より、
\begin{eqnarray} f_{1}h^{2}&=&x^{c}(1-x)^{-a+b-c+1}(1-x)^{2a}\\ &=&x^{c}(1-x)^{a+b-c+1}\\ &=&f_{2} \end{eqnarray}
\begin{eqnarray} g_{1}h^{2}-(f_{1}h^{'})^{'}h&=&-a(c-b)x^{c-1}(1-x)^{a+b-c-1}-ax^{c-1}(1-x)^{a+b-c-1}(bx-c)\\ &=&abx^{c-1}(1-x)^{a+b-c}\\ &=&g_{2} \end{eqnarray}
初期条件検査
\begin{eqnarray} \left\{ \begin{array}{l} F_{1}(0)=(hF_{2})(0)=1\\ F^{'}_{1}(0)=(hF_{2})^{'}(0)=\frac{a(b-c)}{c} \end{array} \right. \end{eqnarray}

\begin{eqnarray} \left\{ \begin{array}{l} (1+x)^{a}{}_{2}F_{1}(\frac{a}{2},\frac{a-b+1}{2};\frac{b+1}{2}|x^{2})={}_{2}F_{1}(\frac{a}{2},\frac{b}{2};b|1-(\frac{1-x}{1+x})^{2})\\ (1+2x)^{a}{}_{2}F_{1}(\frac{a}{3},\frac{a+1}{3};\frac{a+5}{6}|x^{3})={}_{2}F_{1}(\frac{a}{3},\frac{a+1}{3};\frac{a+1}{2}|1-(\frac{1-x}{1+2x})^{3})\\ (1+3x)^{\frac{a}{2}}{}_{2}F_{1}(\frac{a}{4},\frac{a+2}{4};\frac{a+5}{6}|x^{2})={}_{2}F_{1}(\frac{a}{4},\frac{a+2}{4};\frac{a+2}{3}|1-(\frac{1-x}{1+3x})^{2}) \end{array} \right. \end{eqnarray}

A.$(r,s)=(2,2),\rho=1+x$
\begin{eqnarray} \left\{ \begin{array}{l} z=\frac{(1-x)^{2}}{\rho^{2}}\\ 1-z=\frac{4x}{\rho^{2}}\\ z^{'}=-4\frac{1-x}{\rho^{3}} \end{array} \right. \end{eqnarray}
\begin{eqnarray} \left\{ \begin{array}{l} h=(1+x)^{a}=\rho^{a}\\ F_{1}={}_{2}F_{1}(\frac{a}{2},\frac{b}{2};b|1-(\frac{1-x}{1+x})^{2})\\ F_{2}=(1+x)^{a}{}_{2}F(\frac{a}{2},\frac{a-b+1}{2};\frac{b+1}{2}|x^{2}) \end{array} \right. \end{eqnarray}
\begin{eqnarray} \left\{ \begin{array}{l} \mathscr{D}_{1}=\partial x^{b}(1-x)^{a-b+1}\rho^{-a-b+1}-abx^{b-1}(1-x)^{a-b+1}\rho^{-a-b-1}\\ \quad \ =\partial x^{b}(1-x^{2})^{a-b+1}\rho^{-2a}-abx^{b-1}(1-x^{2})^{a-b+1}\rho^{-2a-2}\\ \mathscr{D}_{2}=\partial x^{b}(1-x^{2})^{a-b+1}\partial-a(a-b+1)x^{b}(1-x^{2})^{a-b} \end{array} \right. \end{eqnarray}
\begin{eqnarray} f_{1}h^{2}&=&x^{b}(1-x^{2})^{a-b+1}\\ &=&f_{2} \end{eqnarray}
\begin{eqnarray} (f_{1}h^{'})^{'}&=&\frac{ax^{b-}(1-x^{2})^{a-b+1})}{(1+x)^{a-1}}(\frac{b}{x}-\frac{2(a-b+1)x}{1-x^{2}}-\frac{a+1}{1+x}) \end{eqnarray}
\begin{eqnarray} (f_{1}h^{'})^{'}h&=&ax^{b-1}(1-x^{2})^{a-b}\rho^{-1}(-(a-b+1)x^{2}-(a+1)x+b) \end{eqnarray}
\begin{eqnarray} g_{1}h^{2}-(f_{1}h^{'})^{'}h&=&ax^{b-1}(1-x^{2})^{a-b+1}\rho^{-2}+ax^{b-1}(1-x^{2})^{a-b}\rho^{-1}((a-b+1)x^{2}+(a+1)x-b)\\ &=&ax^{b-1}(1-x^{2})^{a-b}\rho^{-2}(a-b+1)x(1+x)^{2}\\ &=&a(a-b+1)x^{b}(1-x^{2})^{a-b}\\ &=&g_{2} \end{eqnarray}
初期条件検査
\begin{eqnarray} \left\{ \begin{array}{l} F_{1}(0)=F_{2}(0)=1\\ F^{'}_{1}(0)=(hF_{2})^{'}(0)=a \end{array} \right. \end{eqnarray}
B.$(r,s)=(3,3),\rho=1+2x$とする。
\begin{eqnarray} \left\{ \begin{array}{l} h=(1+2x)^{a}=\rho^{a}\\ F_{1}={}_{2}F_{1}(\frac{a}{3},\frac{a+1}{3};\frac{a+1}{2}|1-(\frac{1-x}{1+2x})^{3})\\ F_{2}={}_{2}F_{1}(\frac{a}{3},\frac{a+1}{3};\frac{a+5}{6}|x^{3}) \end{array} \right. \end{eqnarray}
\begin{eqnarray} \left\{ \begin{array}{l} \mathscr{D}_{1}=\partial x^{\frac{a+1}{2}}(1-x^{3})^{\frac{a+1}{2}}\rho^{-2a}\partial-a(a+1)x^{\frac{a-1}{2}}(1-x)^{2}(1-x^{3})^{\frac{a-1}{2}}\rho^{-2a-2}\\ \mathscr{D}_{2}=\partial x^{\frac{a+1}{2}}(1-x^{3})^{\frac{a+1}{2}}\partial-a(a+1)x^{\frac{a+3}{2}}(1-x^{3})^{\frac{a-1}{2}} \end{array} \right. \end{eqnarray}
\begin{eqnarray} f_{1}h^{2}&=&x^{\frac{a+1}{2}}(1-x^{3})^{\frac{a+1}{2}}\\ &=&f_{2} \end{eqnarray}
\begin{equation} (f_{1}h^{'})^{'}=a\frac{(a+1)x^{\frac{a+1}{2}}(1-x^{3})^{\frac{a+1}{2}}}{(1+2x)^{a+1}}(\frac{1}{2x}-\frac{3x^{2}}{2(1-x^{3})}-\frac{2}{(1+2x)}) \end{equation}
\begin{equation} (f_{1}h^{'})^{'}h=a(a+1)x^{\frac{a-1}{2}}(1-x^{3})^{\frac{a-1}{2}}\rho^{-2}(-4x^{4}-4x^{3}-2x+1) \end{equation}
\begin{eqnarray} g_{1}h^{2}-(f_{1}h^{'})^{'}h&=&a(a+1)x^{\frac{a-1}{2}}(1-x)^{2}(1-x^{3})^{\frac{a-1}{2}}\rho^{-2}\\ &+&a(a+1)x^{\frac{a-1}{2}}(1-x^{3})^{\frac{a-1}{2}}\rho^{-2}(4x^{4}+4x^{3}+2x-1)\\ &=&a(a+1)x^{\frac{a+3}{2}}(1-x^{3})^{\frac{a-1}{2}}\\ &=&g_{2} \end{eqnarray}
初期条件検査
\begin{eqnarray} \left\{ \begin{array}{l} F_{1}(0)=hF_{2}(0)=1\\ F^{'}_{1}(0)=(hF_{2})^{'}(0)=2a \end{array} \right. \end{eqnarray}
C.$(r,s)=(4,2),\rho=1+3x$とする。
\begin{eqnarray} \left\{ \begin{array}{l} h=(1+3x)^{\frac{a}{2}}=\rho^{\frac{a}{2}}\\ F_{1}={}_{2}F_{1}(\frac{a}{4},\frac{a+2}{4};\frac{a+2}{3}|1-(\frac{1-x}{1+3x})^{2})\\ F_{2}={}_{2}F_{1}(\frac{a}{4},\frac{a+2}{4};\frac{a+5}{6}|x^{2}) \end{array} \right. \end{eqnarray}
\begin{eqnarray} \left\{ \begin{array}{l} \mathscr{D}_{1}=\partial x^{\frac{a+2}{3}}(1-x^{2})^{\frac{a+2}{3}}\rho^{-a}-\frac{a(a+2)}{2}x^{\frac{a-1}{3}}(1-x)(1-x^{2})^{\frac{a-1}{3}}\rho^{-a-2}\\ \mathscr{D}_{2}=\partial x^{\frac{a+2}{3}}(1-x^{2})^{\frac{a+2}{3}}\partial-\frac{a(a+2)}{4}x^{\frac{a+2}{3}}(1-x^{2})^{\frac{a-1}{3}} \end{array} \right. \end{eqnarray}
\begin{eqnarray} f_{1}h^{2}&=&x^{\frac{a+2}{3}}(1-x^{2})^{\frac{a+2}{3}}\\ &=&f_{2} \end{eqnarray}
\begin{equation} f_{1}h^{'}=\frac{3a}{2}x^{\frac{a+2}{3}}(1-x^{2})^{\frac{a+2}{3}}(1+3x)^{-\frac{a+2}{2}} \end{equation}
\begin{equation} (f_{1}h^{'})^{'}=\frac{3a(a+2)}{2}x^{\frac{a+2}{3}}(1-x^{2})^{\frac{a+2}{3}}\rho^{-\frac{a+2}{2}}(\frac{1}{3x}-\frac{2x}{3(1-x^{2})}-\frac{3}{2(1+3x)}) \end{equation}
\begin{eqnarray} (f_{1}h^{'})^{'}h&=&\frac{a(a+2)}{4}x^{\frac{a-1}{3}}(1-x^{2})^{\frac{a-1}{3}}\rho^{-2}(-9x^{3}-6x^{2}-3x+2) \end{eqnarray}
\begin{eqnarray} g_{1}h^{2}-(f_{1}h^{'})^{'}h&=&\frac{a(a+2)}{4}x^{\frac{a+2}{3}}(1-x^{2})^{\frac{a-1}{3}}\\ &=&g_{2} \end{eqnarray}
初期条件検査
\begin{eqnarray} \left\{ \begin{array}{l} F_{1}(0)=hF_{2}(0)=1\\ F^{'}_{1}(0)=(hF_{2})^{'}(0)=\frac{3a}{2} \end{array} \right. \end{eqnarray}

\begin{eqnarray} K(x)&=&\int_{0}^{\frac{\pi}{2}}\frac{d\theta}{\sqrt{1-x^{2}\cos^{2}\theta}}\\ &=&\frac{\pi}{2}{}_{2}F_{1}(\frac{1}{2},\frac{1}{2};1|x^{2}) \end{eqnarray}

\begin{eqnarray} \left\{ \begin{array}{l} \frac{1}{\sqrt{1-x}}=\sum_{n=0}^{\infty}\frac{(2n)!}{2^{2n}(n!)^{2}}x^{n}\\ \int_{0}^{\frac{\pi}{2}}\cos^{2n}\theta d\theta=\frac{(2n)!}{2^{2n}n!}\frac{\pi}{2}\\ (\frac{1}{2})_{n}=\frac{(2n)!}{2^{2n}n!} \end{array} \right. \end{eqnarray}
を用いればよい。
\begin{equation} \frac{(\frac{1}{2})_{n}(\frac{1}{2})_{n}}{(1)_{n}}=\frac{((2n)!)^{2}}{2^{4n}(n!)^{3}} \end{equation}

\begin{eqnarray} K(x)&=&\frac{\pi}{2}{}_{2}F_{1}(\frac{1}{2},\frac{1}{2};1|x^{2})\\ &=&\frac{\pi}{2(1+x)}{}_{2}F_{1}(\frac{1}{2},\frac{1}{2};1|\frac{4x}{(1-x)^{2}}) \end{eqnarray}

差分と加速

数列$\{a_{n}\}_{n}$に関して次の演算子$\Delta_{ab}$を一般化差分と呼ぶことにする。
\begin{equation} \Delta_{ab}a_{n}=aa_{n}+ba_{n+1} \end{equation}

\begin{eqnarray} \left\{ \begin{array}{l} \Delta_{ab}^{0}a_{n}=a_{n}\\ \Delta_{ab}^{k}a_{n}=\Delta_{ab}(\Delta_{ab}^{k-1}a_{n})\quad(k\in\mathbb{N}) \end{array} \right. \end{eqnarray}
の様に定義すると、次の式が成り立つ。
\begin{equation} \Delta_{ab}^{k}a_{n}=\sum_{l=0}^{k}{}_{k}C_{l}a^{k-l}b^{l}a_{n+l} \end{equation}

$k=1$の場合は明らか。
$1,2,...,k-1$場合まで成り立つと仮定する。次に$k$の場合を考える。
\begin{eqnarray} \Delta_{ab}^{k}a_{n}&=&\Delta(\sum_{l=0}^{k-1}{}_{k-1}C_{l}a^{l}b^{k-1-l}a_{n+l})\\\ &=&{}_{k-1}C_{0}a^{k}b^{0}a_{n}+{}_{k-1}C_{1}a^{k-1}b^{1}a_{n+1}+{}_{k-1}C_{2}a^{k-2}b^{2}a_{n+2}+\cdots +{}_{k-1}C_{k-1}a^{1}b^{k-1}a_{n+k-2}\\ &&\quad\quad\quad\quad\quad\ \ + {}_{k-1}C_{0}a^{k-1}b^{1}a_{n+1}+{}_{k-1}C_{1}a^{k-2}b^{2}a_{n+2}+\cdots+{}_{k-1}C_{k-2}a^{1}b^{k-1}a_{n+k-2}+{}_{k-1}C_{k-1}a^{0}b^{k}a_{n+k-1}\\ &=&a^{k}b^{0}+\sum_{l=1}^{k-1}{}_{k}C_{l}a^{k-1}b^{l}a_{n+l}+a^{0}b^{k}a_{n+k}\\ &=&\sum_{l=0}^{k}{}_{k}C_{l}a^{k-l}b^{l}a_{n+l} \end{eqnarray}

\begin{eqnarray} \left\{ \begin{array}{l} f{(x)}=\sum_{n=0}^{\infty}A_{n}x^{n}\\ f^{(l)}(x)=\sum_{n=0}^{\infty}\frac{n!A_{n}}{(n-l)!}x^{n-l}=l!\sum_{n=l}^{\infty}{}_{n}C_{l}A_{n}x^{n-l} \end{array} \right. \end{eqnarray}
とすると次式が成り立つ。
\begin{eqnarray} f(\lambda\Delta_{ab})a_{0}&=&\sum_{k=0}^{\infty}A_{k}\lambda^{k}\Delta_{ab}^{k}a_{0}\\ &=&\sum_{k=0}^{\infty}A_{k}\lambda^{k}\sum_{l=0}^{k}a^{k-l}b^{l}{}_{k}C_{l}a_{l}\\ &=&\sum_{l=0}^{\infty}\frac{(\lambda b)^{l}a_{l}}{l!}l!\sum_{k=l}^{\infty}{}_{k}C_{l}A_{k}(\lambda a)^{k-l}\\ &=&\sum_{l=0}^{\infty}\frac{(\lambda b)^{l}a_{l}}{l!}f^{(l)}(\lambda a) \end{eqnarray}

$f(x)=\frac{1}{1-x}$とすると$f^{(l)}(x)=\frac{l!}{(1-x)^{l+1}}$なので
\begin{eqnarray} \frac{1}{1-\lambda\Delta_{ab}}a_{0}&=&\sum_{k=0}^{\infty}(\lambda\Delta_{ab})^{k}a_{0}\\ &=&\sum_{l=0}^{\infty}\frac{(\lambda b)^{l}a_{l}}{(1-\lambda a)^{l+1}} \end{eqnarray}
$-a=b=1,\lambda=-\frac{1}{2}$とすると下記の式を得る。
\begin{equation} \sum_{k=0}^{\infty}(-\frac{\Delta}{2})^{k}a_{0}=2\sum_{l=0}^{\infty}(-1)^{l}a_{l} \end{equation}
すなわち、下記の式を得る。
\begin{equation} \sum_{l=0}^{\infty}(-1)^{l}a_{l}=\frac{1}{2}\sum_{k=0}^{\infty}(-\frac{\Delta}{2})^{k}a_{0} \end{equation}

$f(x)=-\log{(1-x)}=\sum_{n=1}^{\infty}\frac{x^{n}}{n}$とすると$f^{(n)}(x)=\frac{(n-1)!}{(1-x)^{n}}$
\begin{eqnarray} -\log{(1-\lambda\Delta_{ab})}a_{0}&=&\sum_{n=1}^{\infty}\frac{(\lambda\Delta_{ab})^{n}}{n}a_{0}\\ &=&\sum_{l=1}^{\infty}\frac{a_{l}}{l}(\frac{\lambda b}{(1-a)})^{l} \end{eqnarray}

数列変換

数列$\{s_{n}\}_{n\in\mathbb{N}}$から数列$\{t_{n}\}_{n\in\mathbb{N}}$への写像$T:\{s_{n}\}\mapsto \{t_{n}\}$を数列変換と呼ぶ。

Richarson補外

数列変換$T:\{s_{n}\}\mapsto\{t_{n}\}$を次の様に定める。
まず次のような漸化式を数列$\{s_{n}\}$を満たすものとする。
\begin{equation} s_{n+1}-s\sim\lambda(s_{n}-s)\quad(\lambda\neq 1) \end{equation}
$\lambda$を既知とし、$s\rightarrow t_{n}$とする。すると次式が成り立つ。
\begin{equation} t_{n}=\frac{s_{n+1}-\lambda s_{n}}{1-\lambda} \end{equation}
この数列変換$T:\{s_{n}\}\mapsto\{t_{n}\}$をRichardson補外という。

先のRichardson補外の場合で$\lambda$が未知とした場合次の式が成り立つ。
\begin{equation} \lambda=\frac{s_{n+1}-s}{s_{n}-s}=\frac{s_{n+2}-s}{s_{n+1}-s} \end{equation}
この式で$s$について解き、$s\rightarrow t_{n}$の様に置きなおす。
\begin{eqnarray} t_{n}&=&\frac{s_{n}s_{n+2}-s_{n+1}^{2}}{s_{n}-2s_{n+1}+s_{n+2}}\\ &=&s_{n}-\frac{(s_{n+1}-s_{n})^{2}}{s_{n}-2s_{n+1}+s_{n+2}}\\ &=&s_{n}-\frac{(\Delta s_{n})^{2}}{\Delta^{2}s_{n}} \end{eqnarray}
この数列変換$T:\{s_{n}\}\mapsto\{t_{n}\}$$Aitken\Delta^{2}$法という。
ちなみに、簡単な計算から$t_{n}$は次の様に書ける事も分かる。
\begin{eqnarray} t_{n}&=&s_{n+1}-\frac{(s_{n+1}-s_{n})(s_{n+2}-s_{n+1})}{s_{n}-2s_{n+1}+s_{n+2}}\\ &=&s_{n+2}-\frac{(s_{n+2}-s_{n+1})^{2}}{s_{n}-2s_{n+1}+s_{n+2}}\\ &=&s_{n+2}-\frac{(\nabla s_{n+2})^{2}}{\nabla^{2}s_{n+2}} \end{eqnarray}
ただし$\nabla a_{n+1}=a_{n+1}-a_{n}$とした。

Euler-Maclaurin展開

\begin{equation} \sum_{k=m}^{n}f(k)=\int_{m}^{n}f(x)dx+\frac{1}{2}(f(m)+f(n))+\sum_{k=2}^{N}(-1)^{k}b_{k}(f^{(k-1)}(n)-f^{(k-1)}(m))+R_{N} \end{equation}

\begin{eqnarray} \sum_{k=m}^{n-1}\int_{0}^{1}\psi{(x)}f^{'}(x+k)dx&=&\sum_{k=m}^{n-1}([\psi{(x)}f(x+k)]_{0}^{1}-\int_{0}^{1}\psi^{'}{(x)}f(x+k)dx)\\ &=&-\psi{(1)}f(m)+\psi{(0)}f(n)+\sum_{k=m}^{n}(\psi{(1)}-\psi{(0)})f(k)-\sum_{k=m}^{n-1}\int_{0}^{1}\psi^{'}(x)f(x+k)dx \end{eqnarray}
ここで、$\psi(x)=x+a$とおき$-\psi(0)=\psi(1)=\frac{1}{2}$とすると$a=-\frac{1}{2}$
\begin{eqnarray} \sum_{k=m}^{n-1}\int_{0}^{1}\psi(x)f^{'}(x+k)dx&=&-\frac{1}{2}(f(m)+f(n))+\sum_{k=m}^{n}f(k)-\sum_{k=m}^{n-1}\int_{0}^{1}f(x+k)dx\\ &=&-\frac{1}{2}(f(m)+f(n))+\sum_{k=m}^{n}f(k)-\int_{m}^{n}f(x)dx\\ \end{eqnarray}
よって、下記の様に書ける。
\begin{equation} \sum_{k=m}^{n}f(k)=\int_{m}^{n}f(x)dx+\frac{1}{2}(f(m)+f(n))+\sum_{k=m}^{n-1}\int_{0}^{1}\psi(x)f^{'}(x+k)dx \end{equation}
ここで次のような記号を定める。
\begin{eqnarray} \left\{ \begin{array}{l} R_{1}=\sum_{k=m}^{n-1}\int_{0}^{1}\psi(x)f^{'}(x+k)dx\\ R_{N}=(-1)^{N-1}\sum_{k=m}^{n-1}\int_{0}^{1}B_{N}(x)f^{(N)}(x+k)dx\\ B_{1}(x)=\psi{(x)}\\ B_{N}(0)=B_{N}(1)=b_{N}\\ B_{N}^{'}(x)=B_{N-1}(x)\\ \end{array} \right. \end{eqnarray}
すると下記の式を得る。
\begin{equation} R_{N}=(-1)^{N-1}b_{N}(f^{N-1}(n)-f^{N-1}(m))+R_{N-1} \end{equation}
ゆえに、以下の式を得る。
\begin{eqnarray} R_{1}&=&b_{2}(f^{(1)}(n)-f^{(1)}(m))+R_{2}\\ &=&b_{2}(f^{(1)}(n)-f^{(1)}(m))+b_{3}(f^{(2)}(n)-f^{(2)}(m))+R_{3}\\ &=&\sum_{k=2}^{N}(-1)^{k}b_{k}(f^{(k-1)}(n)-f^{(k-1)}(m))+R_{N} \end{eqnarray}
つまり、
\begin{equation} \sum_{k=m}^{n}f(k)=\int_{m}^{n}f(x)dx+\frac{1}{2}(f(m)+f(n))+\sum_{k=2}^{N}(-1)^{k-1}b_{k}(f^{(k-1)}(n)-f^{(k-1)}(m))+R_{N} \end{equation}
ここで次のような$B_{k}(x)$の母関数$F(t,x)=1+\sum_{k=1}^{\infty}B_{k}(x)t^{k}$とおく。すると次の様な計算ができる。
\begin{eqnarray} &&\frac{\partial F}{\partial x}=tF\\ &&\log{F}=tx+f(t)\\ &&F=g(t)e^{tx} \end{eqnarray}
また、
\begin{eqnarray} &&\left\{ \begin{array}{l} F(t,0)=-\frac{1}{2}t+\sum_{k=2}^{\infty}b_{k}t^{k}=g(t)\\ F(t,1)=\frac{1}{2}t+\sum_{k=2}^{\infty}b_{k}t^{k}=g(t)e^{t} \end{array} \right.\\ &&F(t,1)-F(t,0)=t=g(t)(e^{t}-1)\\ &&g(t)=\frac{t}{e^{t}-1}\\ &&F(t,x)=\frac{te^{tx}}{e^{t}-1} \end{eqnarray}
以上から、次の結論を得る。
\begin{eqnarray} &&\frac{te^{t}}{e^{t}-1}=1+\frac{1}{2}t+\sum_{k=2}^{\infty}b_{k}t^{k}\\ &&\frac{t}{2}\frac{e^{t}+1}{e^{t}-1}=1+\sum_{k=2}^{\infty}b_{k}t^{k} \end{eqnarray}
また$\frac{-t}{2}\frac{e^{-t}+1}{e^{-t}-1}=\frac{t}{2}\frac{e^{t}+1}{e^{t}-1}$より偶数関。ゆえに、$b_{3}=b_{5}=\cdots=0$
さらにtについて展開し、$t$の同じべきで比較すると下記の式を得る。
\begin{equation} 2(\frac{1}{(2n)!}+\frac{b_{2}}{(2n-2)!}+\cdots+\frac{b_{2n-2}}{2!})=\frac{1}{(2n-1)!}\quad(n=2,3,...) \end{equation}

\begin{eqnarray} \left\{ \begin{array}{l} b_{2}=\frac{1}{12}\\ b_{4}=-\frac{1}{720} \end{array} \right. \end{eqnarray}

Euler-Maclaurin展開

\begin{equation} \sum_{k=m}^{n}f(k)=\int_{m}^{n}f(x)dx+\frac{1}{2}(f(m)+f(n))+\sum_{k=1}^{N}b_{2k}(f^{(2k-1)}(n)-f^{(2k-1)}(m))+R_{2N} \end{equation}

$f(x)=x^{-s}$とすると
\begin{eqnarray} &&\sum_{k=1}^{\infty}\frac{1}{k^{s}}=\frac{1}{s-1}+\frac{1}{2}+\sum_{k=1}^{N}b_{2k}\frac{\Gamma{(s+2k)}}{\Gamma{(s)}}+R_{2N}\\ &&R_{2N}=(-1)^{2N-1}\frac{\Gamma{(s+2N)}}{\Gamma{(s)}}\sum_{k=1}^{\infty}\int_{0}^{1}B_{2N}(x)\frac{1}{(x+k)^{s+2N}}dx\rightarrow 0\quad(N\rightarrow \infty) \end{eqnarray}
\begin{eqnarray} \left\{ \begin{array}{l} \zeta{(s)}=\frac{1}{s-1}+\frac{1}{2}+\sum_{k=1}^{\infty}b_{2k}\frac{\Gamma{(s+2k)}}{\Gamma{(s)}}\quad(1\lt s)\\ b_{2(n-1)}=\frac{1}{(2n-1)!}-2(\frac{1}{(2n)!}+\frac{b_{2}}{(2(n-1))!}+\cdots+\frac{b_{2(n-2)}}{4!}) \end{array} \right. \end{eqnarray}

望遠鏡和

望遠鏡和

数列$\{a_{n}\}_{n\in\mathbb{N}},\{b_{n}\}_{n\in\mathbb{N}}$$a_{n}=b_{n}-b_{n+1}\quad(n\in\mathbb{N})$を満たすとする。すると下記の事実が成り立つ。
\begin{equation} S_{n}=\sum_{k=1}^{n}a_{n}=b_{1}-b_{n+1} \end{equation}
さらに、$\lim_{n\rightarrow \infty}|b_{n}|=0$ならば下記の式が成り立つ。
\begin{equation} \lim_{n\rightarrow \infty}S_{n}=\sum_{k=1}^{\infty}a_{k}=b_{1} \end{equation}
上記の事実を望遠鏡和という。

$\sum_{n=1}^{\infty}\frac{1}{an^{2}+b}$について、$a,b$を適切に定めることで望遠鏡和が成り立つようにせよ。
またその値について考察せよ。
\begin{eqnarray} \sum_{n=1}^{\infty}\frac{1}{an^{2}-b^{2}}&=&\frac{1}{2b}\sum_{n=1}^{\infty}(\frac{1}{an-b}-\frac{1}{an+b})\\ &=&\frac{1}{2b}(\frac{1}{a-b}-\frac{1}{a+b}\\ &&\quad\quad\quad\quad\ \ +\frac{1}{2a-b}-\frac{1}{2a+b}\\ &&\quad\quad\quad\quad\ \quad\quad\quad\quad\ \ +\frac{1}{3a-b}-\frac{1}{3a+b}\\ &&\quad\quad\quad\quad\ \quad\quad\quad\quad\ \quad\quad\quad\quad\ \ +\cdots) \end{eqnarray}
より$na+b=(n+1)a-b$を満たすように$a,b$を定めれば$a=2b$。すると
\begin{equation} \sum_{n=1}^{\infty}\frac{1}{4b^{2}n^{2}-b^{2}}=\frac{1}{2b^{2}} \end{equation}
すなわち、
\begin{equation} \sum_{n=1}^{\infty}\frac{1}{4n^{2}-1}=\frac{1}{2} \end{equation}

Kummer変換

数列$\{a_{n}\}_{n\in\mathbb{N}},\{b_{n}\}_{n\in\mathbb{N}}$について$A=\sum_{n=1}^{\infty}a_{n},B=\sum_{n=1}^{\infty}b_{n}、\gamma=\lim_{n\rightarrow\infty}\frac{b_{n}}{a_{n}}$とする。すると
\begin{eqnarray} A&=&\gamma B+\sum_{n=1}^{\infty}(a_{n}-\gamma b_{n})\\ &=&\gamma B+\sum_{n=1}^{\infty}(1-\gamma\frac{b_{n}}{a_{n}})a_{n} \end{eqnarray}

\begin{eqnarray} 1-(\frac{1}{3}-\frac{1}{5})-(\frac{1}{7}-\frac{1}{9})-\cdots&=&1-2\sum_{k=1}^{\infty}\frac{1}{(4k-1)(4k+1)}\\ &=&1-2\sum_{k=1}^{\infty}\frac{1}{16k^{2}-1}\\ &=&1-2A\\ &=&\frac{\pi}{4} \end{eqnarray}
また
\begin{equation} B=\sum_{n=1}^{\infty}\frac{1}{4n^{2}-1}=\frac{1}{2} \end{equation}
このとき、$\gamma=\lim_{n\rightarrow\infty}\frac{\frac{1}{16n^{2}-1}}{\frac{1}{4n^{2}-1}}=\frac{1}{4}$
Kummer変換を用いると下記の式を得る。
\begin{eqnarray} A&=&\frac{1}{4}\frac{1}{2}+\sum_{n=1}^{\infty}(1-\frac{1}{4}\frac{\frac{1}{4n^{2}-1}}{\frac{1}{16n^{2}-1}})\frac{1}{16n^{2}-1}\\ &=&\frac{1}{8}-\frac{3}{4}\sum_{n=1}^{\infty}\frac{1}{(16n^{2}-1)(4n^{2}-1)}\\ \end{eqnarray}
この式より以下の様な結果を得る。
\begin{eqnarray} \pi&=&4-8A\\ &=&3+6\sum_{n=1}^{\infty}\frac{1}{(16n^{2}-1)(4n^{2}-1)} \end{eqnarray}

Wilf–Zeilberger pair

関数$F(n,k),G(n,k)$をWZ-pairとすると下記の式を満たす。
\begin{equation} F(n+1,k)-F(n,k)=G(n,k+1)-G(n,k)\\ \end{equation}

\begin{equation} \sum_{k=a}^{b}[F(n+1,k)-F(n,k)]=G(n,b+1)-G(n,a) \end{equation}

\begin{eqnarray} \sum_{k=a}^{b}[F(n+1,k)-F(n,k)]&=&\sum_{k=a}^{b}[G(n,k+1)-G(n,k)]\\ &=&-G(n,a)+G(n,a+1)\\ &&\quad\quad\quad\quad\ \ -G(n,a+1)+G(n,a+2)\\ &&\quad\quad\quad\quad\ \ \quad\quad\quad\quad\quad\ \ \ \ -\cdots\\ &&\quad\quad\quad\quad\ \ \quad\quad\quad\quad\quad\ \ \ \ -G(n,b)+G(n,b+1)\\ &=&G(n,b+1)-G(n,a) \end{eqnarray}

Gosperのアルゴリズム

記号

\begin{eqnarray} \left\{ \begin{array}{l} \mathbb{P}[n]=\{f(n)=\sum_{k=0}^{r}a_{k}n^{k}|a_{0},a_{1},...,a_{r}\in\mathbb{C},r\in \{0\}\cup\mathbb{N}\}\\ \mathbb{K}[n]=\{\frac{f(n)}{g(n)}|f(n),g(n)\in\mathbb{K}[n]\} \end{array} \right. \end{eqnarray}

超幾何数列

$t(n)$が超幾何数列であるとは、$\frac{t(n+1)}{t(n)}$がnに関する有理関数$r(n)$になるときをいう。

超幾何数列の名前の由来。

$r(n)$$n$に関する有理関数なので、ある$f(n),g(n)$$n$の多項式を用いて$r(n)=\frac{f(n)}{g(n)}$と書ける。
ゆえに$f(n),g(n)$の根をそれぞれ$a_{1},a_{2},...,a_{k};b_{1},b_{2},...,b_{l}$とすると
\begin{eqnarray} \left\{ \begin{array}{l} f(n)=(n-a_{1})(n-a_{2})\cdots (n-a_{k})\\ g(n)=(n-b_{1})(n-b_{2})\cdots (n-b_{l}) \end{array} \right. \end{eqnarray}
とかけるので、次の様に書ける。
\begin{eqnarray} t(n)&=&\frac{f(n)}{g(n)}t(n-1)\\ &=&\frac{f(n-1)f(n-2)\cdots f(0)}{g(n-1)g(n-2)\cdots g(0)}t(0)\\ &=&\frac{(-a_{1})_{n}(-a_{2})_{n}\cdots (-a_{k})_{n}}{(-b_{1})_{n}(-b_{2})_{n}\cdots (-b_{l})_{n}}t(0) \end{eqnarray}
これは、超幾何級数の係数と同じ性質を持っている。

超幾何数列全体のなす集合を$\mathcal{H}$$\mathcal{H}$によって生成される線形空間を$\mathcal{L}(\mathcal{H})$とおく。

閉形式

数列$f(n)$が閉形式であるとは、$f(n)\in\mathcal{L}(\mathcal{H})$であることを言う。つまり下記の様に書ける事をいう。すなわち、$A_{1}(n),A_{2}(n),...,A_{r}(n)\in\mathcal{H}$が存在して次の様に書ける事だ。
\begin{equation} f(n)=\sum_{k=1}^{r}A_{k}(n) \end{equation}

相似

超幾何数列$A(n),B(n)\in\mathcal{H}$が相似であるとは、その比$\frac{A(n)}{B(n)}$$n$について有理関数になる事をいう。$A(n),B(n)$が相似であるとき、$A(n)\sim B(n)$と書く。

$\sim$」は同値関係を満たす。

[反射率]$A(n)\in\mathcal{H}$とすると$\frac{A(n)}{A(n)}=1\in\mathbb{K}[n]$なので$A(n)\sim A(n)$
[対象律]
\begin{eqnarray} A(n)\sim B(n) &\Leftrightarrow&\frac{A(n)}{B(n)}=\frac{f(n)}{g(n)}\in\mathbb{K}[n]\\ &\Leftrightarrow&\frac{B(n)}{A(n)}=\frac{g(n)}{f(n)}\in\mathbb{K}[n]\\ &\Leftrightarrow&B(n)\sim A(n) \end{eqnarray}
[推移律]
\begin{eqnarray} A(n)\sim B(n)\wedge B(n)\sim C(n)&\Leftrightarrow&\frac{A(n)}{B(n)}=\frac{f_{1}(n)}{g_{1}(n)}\in\mathbb{K}[n]\wedge\frac{B(n)}{C(n)}=\frac{f_{2}(n)}{g_{2}(n)}\in\mathbb{K}[n]\\ &\Leftrightarrow&\frac{A(n)}{B(n)}\frac{B(n)}{C(n)}=\frac{A(n)}{C(n)}=\frac{f_{1}(n)f_{2}(n)}{g_{1}(n)g_{2}(n)}\in\mathbb{K}[n]\\ &\Leftrightarrow&A(n)\sim C(n) \end{eqnarray}

\begin{equation} A(n)\sim B(n)\Leftrightarrow A(n)+B(n)\in \mathcal{H}\vee A(n)+B(n)=0 \end{equation}

[$\Rightarrow$]前提より$f(n),g(n)\in\mathbb{P}[n]$とし$\frac{A(n)} {B(n)}=\frac{f(n)}{g(n)}$
(1)$B(n)=-A(n)$とすると、$A(n)+B(n)=0$
(2)$B(n)\neq -A(n)$とすると
\begin{eqnarray} A(n)+B(n)&=&(\frac{f(n)}{g(n)}+1)B(n)\\ &=&\frac{h(n)}{g(n)}B(n) \end{eqnarray}
ゆえに
\begin{eqnarray} \frac{A(n+1)+B(n+1)}{A(n)+B(n)}&=&\frac{h(n+1)g(n)}{g(n+1)h(n)}\frac{B(n+1)}{B(n)}\in\mathbb{R}[n] \end{eqnarray}
このことから$A(n)+B(n)\in\mathcal{H}$
[$\Leftarrow$]
(1)$A(n)+B(n)=0$の場合は$\frac{A(n)}{B(n)}=-1\in\mathbb{K}[n]$より$A(n)\sim B(n)$
(2)$A(n)+B(n)\in\mathcal{H}$である場合は、$R_{A}=\frac{A(n+1)}{A(n)}$
\begin{eqnarray} \left\{ \begin{array}{l} R_{A}=\frac{A(n+1)}{A(n)}\in\mathbb{K}[n]\\ R_{B}=\frac{B(n+1)}{B(n)}\in\mathbb{K}[n]\\ R_{A+B}=\frac{A(n+1)+B(n+1)}{A(n)+B(n)}\in\mathbb{K}[n]\\ R_{\frac{A}{B}}=\frac{A(n)}{B(n)} \end{array} \right. \end{eqnarray}
とすると
\begin{eqnarray} &&R_{A+B}=\frac{R_{\frac{A}{B}}R_{A}+R_{B}}{R_{\frac{A}{B}}+1}\\ &&R_{\frac{A}{B}}=-\frac{R_{A+B}-R_{B}}{R_{A+B}-R_{A}}\in\mathbb{K}[n] \end{eqnarray}

Gosperのアルゴリズム

$t(n),u(n)$をそれぞれ超幾何項で下記の性質を持つものとする。
\begin{equation} u(n+1)-u(n)=t(n) \end{equation}
このとき、$u(n)$を求める手順をGosperのアルゴリズムという。

\begin{equation} \sum_{n=a}^{b}t(n)=u(b+1)-u(a) \end{equation}

\begin{equation} u(n)(\frac{u(n+1)}{u(n)}-1)=t(n) \end{equation}
より、
\begin{eqnarray} \left\{ \begin{array}{l} \frac{t(n+1)}{t(n)}=r(n)\\ u(n)=R(n)t(n) \end{array} \right. \end{eqnarray}
と置きなおせば次式を得る。
\begin{equation} r(n)R(n+1)-R(n)=1 \end{equation}

$a_{1},a_{2},...,a_{k}\in\mathbb{C}$とする。このとき、$a_{1}-1,a_{2}-1,...,a_{k}-1$すると、下記の事実が成り立つ。
『ある$i\in\{1,2,...,k\}$が存在して任意の$j\in\{1,2,...,k\}$に対して$a_{i}\neq a_{j}-1$が成り立つ。』-(★)

[1]まず$a_{1},a_{2},...,a_{k}$を異なる値のグループ$A_{1},A_{2},...,A_{p}$に分ける。ただし、これらグループは下記の様に定めた。
\begin{equation} A_{q}=\{a_{i}|A_{q}=a_{i}\} \end{equation}
[2]仮に、(★)が成り立たないとすると、いかなる$A_{i}$に対しても、$A_{j}-1$が存在して$A_{i}=A_{j}-1$が成り立つ。
[3]このとき、二つの$A_{i},A_{k}$が同じ$A_{j}-1$と一致することはない。このと事は実際
$A_{i}\neq A_{k}$かつ$A_{i}=A_{k}$となり矛盾してしまうことから分かる。
[4]ゆえに、$A_{1}-1,A_{2}-1,...,A_{p}-1$$A_{1},A_{2},...,A_{p}$を並べたものだとわかる。
[5]ゆえに
\begin{eqnarray} &&(A_{1}+A_{2}+\cdots +A_{p})-k=A_{1}+A_{2}+\cdots +A_{k}\\ &&k=0 \end{eqnarray}
[6]しかし複素数が少なくとも一つ以上存在する事から、グループは少なくとも一個以上存在するので$0\lt k$。ゆえに$0=k\wedge 0\lt k$となり矛盾。
以上より(★)は示された。

Gosper方程式

下記の様な変数変換を行う。
\begin{eqnarray} \left\{ \begin{array}{l} r(n)=\frac{a(n)}{b(n)}\frac{c(n+1)}{c(n)}\\ R(n)=\frac{b(n-1)}{c(n)}S(n) \end{array} \right. \end{eqnarray}
この変換により次式を得る。
\begin{equation} a(n)S(n+1)-b(n-1)S(n)=c(n) \end{equation}

\begin{equation} u(n)=\frac{b(n-1)S(n)}{c(n)}t(n) \end{equation}

$t(n)=n^{k}p(n)$とした場合
\begin{equation} \frac{t(n+1)}{t(n)}=\frac{(n+1)^{k}}{n^{k}}\frac{p(n+1)}{p(n)} \end{equation}
より
\begin{eqnarray} \left\{ \begin{array}{l} a(n)=(n+1)^{k}\\ b(n)=n^{k}\\ c(n)=p(n) \end{array} \right. \end{eqnarray}
これからGosper方程式は次の様に書ける。
\begin{equation} (n+1)^{k}S(n+1)-(n-1)^{k}S(n)=p(n) \end{equation}
仮に$S(n)=2^{n}$であるとすると次の様に書ける。
\begin{equation} p(n)=2^{n}(2(n+1)^{k}-(n-1)^{k}) \end{equation}
ゆえに
\begin{eqnarray} u(n)&=&\frac{(n-1)^{k}}{p(n)}2^{n}n^{k}p(n)\\ &=&2^{n}(n(n-1))^{k} \end{eqnarray}
これにより次の事実が分かる。
\begin{equation} \sum_{n=0}^{N}n^{k}2^{n}(2(n+1)^{k}-(n-1)^{k})=2^{N+1}(N(N+1))^{k} \end{equation}

$t(n)=A^{n}\begin{pmatrix}2n\\n\end{pmatrix}p(n)$した場合
\begin{equation} \frac{t(n+1)}{t(n)}=\frac{2A(2n+1)}{n+1}\frac{p(n+1)}{p(n)} \end{equation}
より
\begin{eqnarray} \left\{ \begin{array}{l} a(n)=2A(2n+1)\\ b(n)=n+1\\ c(n)=p(n) \end{array} \right. \end{eqnarray}
ゆえに、Gosper方程式は次の様に書ける。
\begin{equation} 2A(2n+1)S(n+1)-nS(n)=p(n) \end{equation}
$A=\frac{1}{2}$とすると
\begin{equation} (2n+1)S(n+1)-nS(n)=p(n) \end{equation}
$S(n)=1$とすると$p(n)=n+1$
これから、
\begin{eqnarray} u(n)&=&\frac{n}{n+1}\frac{\begin{pmatrix}2n\\n\end{pmatrix}}{2^{n}}(n+1)\\ &=&\frac{n\begin{pmatrix}2n\\n\end{pmatrix}}{2^{n}} \end{eqnarray}
ゆえに
\begin{equation} \sum_{n=0}^{N}\frac{(n+1)\begin{pmatrix}2n\\n\end{pmatrix}}{2^{n}}=\frac{(N+1)\begin{pmatrix}2N+2\\N+1\end{pmatrix}}{2^{N+1}} \end{equation}

$t(n)=A^{n}\begin{pmatrix}2n\\n\end{pmatrix}^{k}p(n)$とした場合
\begin{equation} \frac{t(n+1)}{t(n)}=\frac{A2^{k}(2n+1)^{k}}{(n+1)^{k}}\frac{p(n+1)}{p(n)} \end{equation}
より
\begin{eqnarray} \left\{ \begin{array}{l} a(n)=A2^{k}(2n+1)^{k}\\ b(n)=(n+1)^{k}\\ c(n)=p(n) \end{array} \right. \end{eqnarray}
この事から求めるGosper方程式は次の様になる。
\begin{equation} A2^{k}(2n+1)^{k}S(n+1)-n^{k}S(n)=p(n) \end{equation}
ここで$S(n)=(2n+1)^{k}$とすると
\begin{equation} p(n)=A2^{2k}(n+1)^{k}-n^{k} \end{equation}
特に$A=\frac{1}{2^{2k}}$とすると
\begin{equation} p(n)=(n+1)^{k}-n^{k} \end{equation}
\begin{eqnarray} u(n)&=&\frac{n^{k}}{p(n)}(2n+1)^{k}\frac{\begin{pmatrix}2n\\n\end{pmatrix}^{k}}{2^{2kn}}p(n)\\ &=&\frac{n^{k}(2n+1)^{k}\begin{pmatrix}2n\\n\end{pmatrix}^{k}}{4^{kn}} \end{eqnarray}
これから次のような結果を得る
\begin{equation} \sum_{n=0}^{N}\frac{\begin{pmatrix}2n\\n\end{pmatrix}^{k}}{4^{kn}}((n+1)^{k}-n^{k})=\frac{(N+1)^{k}(2N+3)^{k}\begin{pmatrix}2N+2\\N+1\end{pmatrix}^{k}}{4^{k(N+1)}} \end{equation}

超幾何双和

$F(n,s_{1},...,s_{e})$$s_{1},...,s_{e},n$それぞれに対して超幾何項であるとする。この時次の様な問題を考える。
\begin{equation} \sum_{k=0}^{\gamma}p_{k}(n)S(n+k)=0 \end{equation}
ただし、
\begin{equation} S(n)=\sum_{s_{1}}\cdots\sum_{s_{e}}F(n,s_{1},...,s_{e}) \end{equation}

$e=1$とし$s_{1}=r$と置きなおす。
このとき
\begin{equation} \sum_{k=0}^{\gamma}p_{k}(n)f(n+k,r)=g(n,r+1)-g(n,r) \end{equation}
であるとすると下記の事実が成り立つ。
\begin{equation} \sum_{k=0}^{\gamma}p_{k}(n)S(n+k)=\lim_{R\rightarrow\infty}g(n,R+1)-g(n,0) \end{equation}

参考文献

投稿日:13日前
更新日:9分前

この記事を高評価した人

高評価したユーザはいません

この記事に送られたバッジ

バッジはありません。

投稿者

コメント

他の人のコメント

コメントはありません。
読み込み中...
読み込み中